Find the electric field at the center of square. Assume that q1=11.8nC, q2=-11.8nC, q3=23.6nC, q4=-23.6nC and a=5.2cm. Such that 'a' is the distance between any two charges. The charge q1 and q3 are along two sides of a diagonal.

Answers

Answer 1

Answer:

[tex]E_T=[-27739.6\hat{i}-55479\hat{j}]\frac{N}{C}[/tex]

Explanation:

You have four charges at the corners of a square of side a=5.2cm

In order to calculate the electric field at the center of the square, you sum the contribution of the electric field generated by each charge.

The total electric field is given by:

[tex]E_T=E_1+E_2+E_3+E_4\\\\[/tex]    (1)

each contribution to the total electric field has two components x and y. The signs of the components depends of the direction of the field, which is given by the sign of the charge that produced the electric field. Then, you have

[tex]E_1=k\frac{q_1}{r^2}cos\theta\hat{i}-k\frac{q_1}{r^2}sin\theta\hat{j}\\\\E_1=k\frac{q_1}{r^2}(cos\theta \hat{i}-sin\theta \hat{j})[/tex]   (2)

q1 = 11.8*10^-9 C

k: Coulomb's constant = 8.98*10^9 Nm^2/C^2

For a square you obtain that

[tex]r=\sqrt{2}a=\sqrt{2}(5.2cm)=7.35cm=7.35*10^{-2}m[/tex]

and the angle is 45°

Then, you have in the equation (2):

[tex]E_1=(8.98*10^9Nm^2/C^2)\frac{11.8*10^{-9}C}{(7.35*10^{-2}m)^2}(cos45\° \hat{i}-sin45\° \hat{j})=[13869.7\hat{i}-13869.7\hat{j}]\frac{N}{C}[/tex]

In the same way you obtain for the other contributions to the total electric field:

For E2:

[tex]E_2=k\frac{q_2}{r^2}(cos45\°\hat{i}+sin45\° \hat{j})\\\\E_2=[13869.7\hat{i}+13869.7\hat{j}]\frac{N}{C}[/tex]

For E3:

[tex]E_3=k\frac{q_3}{r^2}(-cos45\°\hat{i}+sin45\°\hat{j})\\\\E_3=(8.98*10^9Nm2/C^2)\frac{23.6*10^{-9}C}{(7.35*10^{-2}m)^2}(-cos45\°\hat{i}+sin45\°\hat{j})\\\\E_3=39229.58(-cos45\°\hat{i}+sin45\°\hat{j})\frac{N}{C}\\\\E_3=[-27739.5\hat{i}+-27739.5\hat{j}]\frac{N}{C}[/tex]

for E4:

[tex]E_4=k\frac{q_4}{r^2}(-cos45\°\hat{i}-sin45\°\hat{j})\\\\E_4=[-27739.5\hat{i}-27739.5\hat{j}]\frac{N}{C}[/tex]

Finally, you sum component by component the four contributions to the total electric field (equation (1)):

[tex]E_T=[-27739.6\hat{i}-55479\hat{j}]\frac{N}{C}[/tex]

Answer 2

There still are four chargers in the corners of a square with side a=5.2cm. To compute the electric field just at the center of the composition, add the contributions of each shield's electromagnetic current.

entire electric field

[tex]\to E_T=E_1+E_2+E_3+E_4\ \ \ \ \ \ \ (1)[/tex]

Every contribution to the total electric field is comprised of two components, x, and y. These components' signs were determined by the direction of the field, which is determined by the sign of the charge that produced the electric field. There's

[tex]\to E_1=k\frac{q_1}{r^2}\cos\theta\hat{i}-k\frac{q_1}{r^2}\sin\theta\hat{j}\\\\\to E_1=k\frac{q_1}{r^2}(\cos\theta \hat{i}-\sin\theta \hat{j}) \ \ \ \ (2)\\\\\to q_1 = 11.8\times 10^{-9}\ C\\\\[/tex]

Coulomb’s constant (K) [tex]= 8.98\times 10^9\ \frac{Nm^2}{C^2}[/tex]

square, you obtain that

[tex]\to r=\sqrt{2}\\\\\to a=\sqrt{2}(5.2\ cm)=7.35\ cm=7.35\times 10^{-2}\ m[/tex]

angle= [tex]45\°[/tex]  then, you have in the equation (2):

[tex]\to E_1=(8.98\times 10^9\ \frac{Nm^2}{C^2})\frac{11.8 \times 10^{-9}\ C}{(7.35 \times 10^{-2}m)^2}(\cos 45^{\circ} \hat{i}-\sin 45^{\circ} \hat{j})[/tex]

         [tex]=[13869.7\hat{i}-13869.7\hat{j}]\ \frac{N}{C}[/tex]

Similarly, for other components to the overall electric field, you get:

For E2:

[tex]\to E_2=k\frac{q_2}{r^2}(\cos45^{\circ}\hat{i}+\sin45^{\circ} \hat{j})\\\\ \to E_2=[13869.7\hat{i}+13869.7\hat{j}]\ \frac{N}{C}[/tex]

For E3:

[tex]\to E_3=k\frac{q_3}{r^2}(-\cos45^{\circ}\hat{i}+\sin45^{\circ} \hat{j})\\\\\to E_3=(8.98\times 10^9 \ \frac{Nm^2}{C^2}) \ \frac{23.6 \times 10^{-9}\ C}{(7.35\times 10^{-2}\ m)^2}(-\cos45^{\circ} \hat{i}+\sin45^{\circ} \hat{j})\\\\\to E_3=39229.58(-\cos45^{\circ} \hat{i}+\sin45^{\circ} \hat{j})\frac{N}{C}\\\\\to E_3=[-27739.5\hat{i}+-27739.5\hat{j}]\ \frac{N}{C}\\[/tex]

For E4:  

[tex]\to E_4=k\frac{q_4}{r^2}(-\cos45^{\circ}\hat{i}-\sin45^{\circ}\hat{j})\\\\ \to E_4=[-27739.5\hat{i}-27739.5\hat{j}]\ \frac{N}{C}[/tex]

Finally, we add up the four contributions to the total electric field (equation (1)):

[tex]E_T=[-27739.6\hat{i}-55479\hat{j}]\frac{N}{C}[/tex]

Learn more:

brainly.com/question/14696059


Related Questions

How can I makeup a physics scenario that includes concepts of work and energy. Should I make something going up or down a ramp!? JUST a SCENARIO plss!!!I'm stressed! I need to be able to make a solution for this made up scenario that includes work and energy calculations.

Answers

Answer:

Work done = 100 Nm

Potential energy = 250J

Explanation:

A person carrying a heavy weight of load without moving does no work. But the load will surely experience potential energy.

Work will be done when he or she covers a little distance.

If the weight of the load = 25 N and the distance covered is 4 metres, the workdone will be the product of the weight and distance covered. That is,

WD = F × d

WD = 25 × 4 = 100Nm

Let assume that the height of the man is 10 m tall.

The load will be experiencing potential energy of mgh. That is,

P.E = mgh

Since weight W = mg

Substitute weight and height into the formula

P.E = 25 × 10 = 250J

In the Position versus Time graph of an object moving in a straight line, what is the change in position in the fourth hour?
A) Zero
B) 15 miles
C) 35 miles
D) 75 miles

Answers

Answer:

Option B. 15miles.

Explanation:

From the question given, in the 4th hour simply means between 3rd and 4th hour.

Now, let us determine the position of the object at the 3rd and 4th hour. This is illustrated below:

At the 3rd hour:

Position (P3) = 60miles

At the 4th hour:

Position (P4) = 75miles

Finally, we shall determine the change in the position of the object as follow:

Change in the position = P4 – P3

Change in the position = 75 – 60

Change in the position = 15 miles

Therefore, the change in the position of the object in the 4th hour is 15miles.

Answer:

its b

Explanation:

i got it right on my exam

describe how light reflecting from a mirror can produce an image. In particular, explain how mirrors can produce images that are larger or smaller than life size, as well as upright or inverted.

Answers

Answer:

A larger image is produced when  [tex]d_i[/tex] > [tex]d_o[/tex]

A smaller image is produced when [tex]d_i[/tex] < [tex]d_o[/tex]

An upright image  is produced when m is positive

An upright image  is produced when m is negative

Explanation:

The mirror equation is given as follows;

[tex]\dfrac{1}{f} = \dfrac{1}{d_i} + \dfrac{1}{d_o}[/tex]

[tex]m =-\dfrac{d_i}{d_o} = \dfrac{h_i}{h_o}[/tex]

For concave mirrors, f = focal length

[tex]d_i[/tex] = Image distance from the mirror (-ve [tex]d_i[/tex] = Image is behind the mirror +ve [tex]d_i[/tex] = Image is in front of the mirror)

[tex]d_o[/tex] = Object distance from the mirror (-ve [tex]d_o[/tex] = Object is behind the mirror +ve [tex]d_o[/tex] = Object is in front of the mirror)

m = Magnification (-ve m = Inverted image +ve m = upright image)

[tex]h_i[/tex] = Image height

[tex]h_o[/tex] = Object height

f = Focal length of the mirror

To produce a larger image [tex]d_i[/tex] > [tex]d_o[/tex]

To produce a smaller image [tex]d_i[/tex] < [tex]d_o[/tex]

To produce an upright image, m should be positive hence, [tex]d_i[/tex] will be negative or the image will appear behind the mirror

To produce an inverted image, m should be negative hence, [tex]d_i[/tex] will be positive or the image will form in front of the mirror.

A student is flying west on a school trip from Winnipeg to Calgary in a jet that has an air velocity of 792 km/h.The direction the plane would have to fly to compensate for a wind velocity of 62.0 km/h [N] is _____° S of W. (give your answer with the correct number of significant digits and do not include units)

Answers

Answer:

The direction the plane would have to fly to compensate for a wind velocity of 62.0 km/h[N] is 4.5° S of W

Explanation:

The given parameters are;

Velocity of Jet = 792 km/h

Direction of jet velocity = West

Velocity of wind = 62.0 km/h

Direction of wind velocity = North

Therefore, the jet has to have a component of 62.0 km/h South of West to compensate for the wind velocity

The direction of the plane, θ° South of West (S of W) to compensate for the wind is given as follows;

[tex]Tan \left (\theta \right )= \dfrac{62}{792} = \dfrac{31}{396}[/tex]

Therefore;

[tex]\theta = tan^{-1}\left (\dfrac{31}{396} \right ) = 4.476^{\circ} \approx 4.5^{\circ}[/tex]

The direction the plane would have to fly to compensate for a wind velocity of 62.0 km/h[N] = 4.5° S of W.

The direction the plane would have to fly to is : 4.5° south of west

Given data :

Jet velocity = 792 km/h

wind velocity = 62 km/h

Given that the direction of the jet velocity is west and the direction of the wind velocity is north.

The direction of component velocity of the Jet would be 62 km/h south of west

Determine the direction ( ∅ ) of the plane south of west

Tan ( ∅ ) = 62 / 792

Therefore ( ∅ ) = Tan⁻¹ ( 62/792 )

                          ≈ 4.5° S of W

Hence we can conclude that The direction the plane would have to fly to is : 4.5° south of west

Learn more about wind velocity : https://brainly.com/question/1358771

In Los Angeles, the temperature at 3:00 p.m on Aug 16th is
73.9 °F. What is the temperature in °C?
°C.

Answers

Answer:

The temperature on 16th August is 23.37 degree Celsius.

Explanation:

In Los Angeles, the temperature at 3:00 p.m on Aug 16th is  73.9 °F. It is required to convert it into degree Celsius.

The conversion from Fahrenheit to Celsius is as follows :

[tex]C=(F-32)\times \dfrac{5}{9}\\\\C=(73.9-32)\times \dfrac{5}{9}\\\\T=23.27^{\circ} C[/tex]

So, the temperature on 16th August is 23.37 degree Celsius.

A boy pushes on a wagon so that it accelerates at a rate of 0.50 m/s2. The wagon has a mass of 24 kg. What is the magnitude of the boy's pushing force? (Ignore frictional effects.) 12 N 12 kg 48 N 48 kg

Answers

Answer:

12N

Explanation:

Force acting on any body is equal to mass of body multiplied by acceleration produced in the body.

Its formula is

F = M*a

where F is the force acting on body

M is the mass of the body

a is the acceleration produced in the body

Unit of force is Newton represented by N

1 N is force acting on body of mass 1 kg which produces acceleration of 1m/s^2.

_____________________________________

Given

a = 0.50 m/s^2

M = 24 kg

therefore

F = 24*0.5 = 12 N

Thus,  magnitude of the boy's pushing force is 12 N.

70 POINTS HELP PHYSICS THE PICTURE HAS THE QUESTION AND POSSIBLE CHOICES

Answers

Answer:

0.3 m/s

Explanation:

0.5 - 0.2 = .3

A dog walks Along the ground. If the dog applies an action force on the ground, what is the reaction force?

Answers

the reaction force is weight .

Answer: The answer is weight.

Explanation:

What is the total resistance of five 10 ohms resistors in life parallel... a)500 ohms
B)2 ohms
C)10 ohms
D)50 ohms?

Answers

Answer: B

R = 2 ohms

Explanation:

When resistors are connected in parallel, the equivalent resistance R will be less than the least resistance.

1/R = 1/R1 + 1/R2 + 1/R3 + 1/R4 + 1/R5

Since the 5 resistors are 10 ohms each, substitute them in to the formula

1/R = 1/10 + 1/10 +1/10 + 1/10 + 1/10

The LCM = 10

1/R = 5/10

Reciprocate both sides

R = 10/5 = 2 ohms

The correct option is B

An object is placed 10 m to the left of a convex lens with a focal length of +8 cm Where is the image of the object

18 cm to the left of the lens
40 cm to the right of the lens
40 cm to the left of the lens
18 cm to the right of the lens

Answers

Answer:

18 cm to the right

Explanation:

I don't really know i'm just guessing. hope it's right.

A 1,500 kg car hits a boulder with a force of 500 N, and the collision takes 0.5
seconds. What is the magnitude of its impulse?
A. 75,000 kg-m/s
B. 50 kg-m/s
C. 250 kg-m/s
D. 750 kg-m/s

Answers

Answer: C.

250 kg-m/s

Explanation:

Given that the

Mass M = 1,500 kg

Force F = 500 N

Time t = 0.5 seconds

From Newton's second law of motion which state that the rate of change of momentum is proportional to the applied force.

F = mV/t

Ft = mV

Where ft = impulse: the product of time and applied force

Substitutes force and time into the formula

Ft = 500 × 0.5 = 250 Ns

Answer:

c

Explanation:

just took the quiz

Lenses work using the _____
of light.
A) emission
B) reflection
C) refraction
D) absorption

Answers

The answer is C. The working of a lens is based on the refraction of light when light passes through it.

How can I makeup a physics scenario that includes concepts of work and energy. Should I make something going up or down a ramp!? JUST a SCENARIO plss!!!I'm stressed! I need to be able to make a solution for this made up scenario that includes work and energy calculations.​

Answers

Answer: Work done = 100Nm

Potential energy = 250 J

Find the remaining solution in the explanation

Explanation:

A person carrying a heavy weight of load without moving does no work. But the load will surely experience potential energy.

Work will be done when he or she covers a little distance.

If the weight of the load = 25 N and the distance covered is 4 metres, the workdone will be the product of the weight and distance covered. That is,

WD = F × d

WD = 25 × 4 = 100Nm

Let assume that the height of the man is 10 m tall.

The load will be experiencing potential energy of mgh. That is,

P.E = mgh

Since weight W = mg

Substitute weight and height into the formula

P.E = 25 × 10 = 250J


How much work must be done on a 10-kg bicycle to increase its speed from 5
m/s to 10 m/s?
O A. 500 J
B. 125 J
C. 375J
O D. 5J

Answers

Answer:

375 j

Explanation:

Work done = increase in kinetic energy

C is the correct answer for this question. 375 work must be done on a 10 kg bicycle.

1:   1/5*10 * (10+5)(10-5)

Work Done: 375J

Hope this helps.

What is the mass of an object that weighs 198 N?

Answers

Answer: An object weighs 2800 lb

Explanation:

An important step toward a more complete understanding of blackbody
radiation occurred when Max Planck developed a theory that presented which
of the following ideas?
A. Light is composed of discrete chunks that must be absorbed as a
unit.
B. Large objects can be viewed as acting as waves.
C. Electrons can act as both waves and particles.
D. Matter is composed of discrete chunks that cannot be broken
down.

Answers

Answer:

The correct option is;

C. Electrons can act as both waves and particles

Explanation:

The quantum theory theory developed by Max Planck established that effect of radiation on a blackbody can be explained by the fact that energy can exhibit physical characteristics which implies that energy can have both wave-like and physical characteristics

With Planck's law, light and electrons are known as having wave particle duality, that is they can act as both waves and particles.

Answer:

The answer is A. Light is composed of discrete chunks that must be absorbed as a unit.

Explanation:

Hope this helps.

What must a magnet always have?
A. Two like poles
B. Two opposite charges
C. Two like charges
D. Two opposite poles​

Answers

Answer:

the anwser is d

Explanation:

you must have a north and south pole

Answer:

your anwser would be

D) two opposite poles

If you are swimming upstream (i.e., against the current), at what speed does your friend on the shore see you moving?

Answers

Could be very slow since they’re basically going against the current which is hard so will be going slow

Assuming that voltage remains constant, what happens to the current in a wire as its diameter decreases? A. The current is not affected by a change in wire diameter. B. The current alternates between high and low values. C. The current increases. D. The current decreases.

Answers

Answer:

The current decreases

Explanation:

A P E X

Assuming that voltage remains constant, as its diameter decreases, its current decreases. hence option D is correct.

What is Current ?

Current is a flow of charges. it is denoted by i and expressed in ampere A. Mathematically it is expressed as i = q/t, where q is the amount of charge and t is time. Current is nothing but amount of charges flown in the unit time in the electric wire. Charge is expressed in coulomb C and time in second s. hence coulomb per second (C/s) is ampere A. Charge on electron is 1.60217663 × 10⁻¹⁹C which is called as elementary charge.

There are two types of the current, Convectional current and non-conventional current. Convectional current is the current flows from positive to negative. Non convectional current flows from negative to positive. Note that flow of electrons is from negative to positive. Hence direction of flow of conventional current is from positive to negative.

In this problem, voltage is constant and diameter of the wire is decreasing,

The relation between resistance and and resistivity is

R = ρ l/A

When diameter decreases, cross sectionals area decreases, it increases resistance of the wire. and due to increase in resistance, current decreases in order to keep the voltage constant(V=IR).

Hence option D is correct.

to know more about Current :

https://brainly.com/question/13076734

#SPJ3.

What is the electric potential 20.0 cm from a negative point charge of 15 µC?

Answers

Answer: V = 675000v

Explanation:

Electric potential V can be calculated by using the formula

V = Kq/r

Where

V = electric potential

q = charge = 15µC

K = Coulomb constant = 9 × 10^9Nm^2C^-2

r = 20 cm

Substitute all the parameters into the formula

V = (9×10^9 × 15×10^-6)/20×10^-2

V = 135000/0.2

V = 675000 volt

The presence of electrons in a substance makes it a conductor of electricity.

True
False

Answers

Answer:

False

Explanation:

Just finished the exam

The presence of 'free electrons' in a substance makes it a conductor of electricity. So, the statement is false.

What is meant by a conductor ?

The objects or substances that let the flow of electricity through them are known as conductors. The ability to carry heat is another benefit of conductors.

Here,

Since, every electron in an atom carries a negative charge, they can serve as our charge carriers. We can produce electricity if we can compel an electron to move by releasing it from its atom.

Current flowing through the conductors is caused by the free electrons present in them.

Free electrons are valence shell electrons with a weaker nucleus-electron interaction; as a result, they are free to travel in the metal, which acts as a conductor, and carry out current flow.

Hence,

The presence of 'free electrons' in a substance makes it a conductor of electricity. So, the statement is false.

To learn more about conductor, click:

https://brainly.com/question/18084972

#SPJ3

what’s the difference between gravitational force & gravitational field strength?

Answers

Answer:

Gravitational field strength is the force experienced by a unit mass. Gravitational force is the amount of force acting on a body. It is the product of field strength times the mass under consideration. Gravitational pull is just a more colloquial name for gravitational force.

Explanation:

hope it helps u

what’s the difference between gravitational force & gravitational field strength?

What’s the a chicken nugget

Answers

It’s Chicken strips ?

Give example of not reversible change...

Answers

Answer:

There were many examples of this. One of the most well-known not reversible reaction is a precipitation reaction, in which an insoluble solid is formed from two aqueous solutions. An example is the reaction between silver nitrate and sodium chloride which forms a silver chloride precipitate.

A rope is vibrating at high frequency. The length of the rope is 2.40 meters. A snapshot of the rope at a given moment in time
is also shown. Use this information to determine the
wavelength of the wave.​

Answers

Answer:

λ = L / # _ periods

Explanation:

We don't have the image of the string, but a wavelength is the minimum distance when the wave repeats, therefore the distance between two consecutive maxima is the wavelength in the string

A method used to decrease the error is to measure the distance in several consecutive maxima and divide the number of them, in this way the error in the measurements is divided between all the points, so the value of the wavelength is more exact

               λ = L / # _ periods

Which statement about magnets and magnetic fields is correct?

A. Magnetic fields apply a force on any object with a mass.

B. Magnetic fields apply a force on any metal.

C. Magnetic fields apply a force on magnets.

D. Magnetic fields apply a force on electrically conductive metals.

Answers

Answer:

B. Magnetic fields apply a force on any metal.

Explanation:

hope this helps u

Answer:

Magnetic fields apply a force on magnets.

Explanation:

this is correct

A rose has a phenotype ratio of 3:1 where red roses are dominant over white roses. Out of a total of 4 roses, how many of the roses are red? 1 2 3 4

Answers

Answer: 3

Explanation:

Since the phenotype ratio is 3 : 1, where red Roses are dominant over white roses, the ratio of red will be 3 while the white will be 1

Finding the value of the two roses will be;

3 + 1 = 4

For red roses,

3/4 × 4 = 3

For white rose,

1/4 × 4 = 1

Therefore, 3 of the red roses are there.

Answer:

3

Explanation:

where was forensic science first used to solve a crime?

Answers

Answer:

In 1836, Scottish chemist, James Marsh, did the first application of this forensic science technique. This test was actually used successfully in a murder trial at that time.

Explanation:

I hope that is what you were asking

Answer: London , United Kingdom

Explanation:

In 1836, Scottish chemist, James Marsh, did the first application of this forensic science technique.

There is a limited amount of fossil fuels (our main source of energy for powering our appliances and cars) on Earth. Why? What is an example of a "renewable" energy source, and why is it considered "renewable"?

Answers

There, is a limited amount of Fossil fuels because, there isn’t a supply of it that will last forever/for the long term. An example, of a renewable energy source would be Solar which has a consistent and a supply that will last for awhile and wont run out any time soon. (However, it isn’t the most efficient power source)

What statement is best supported by the information in the chart?

Answers

Answer:

Object 1

Explanation:

The statement  best supported by the information in the chart is that the object 1 is the heaviest.

What is weight?

Weight is the product of mass of the object and gravitational acceleration of the Earth g =9.81 m/s².

As the balance tips towards the object, the object is heavier than the weight.

Thus, the object 1 is the heaviest.

Learn more about weight.

https://brainly.com/question/10069252

#SPJ2

Other Questions
Why should government regulation hold social media companies responsible for what is posted on their platform? an olympic runner completes the 200 meter sprint in 23 seconds what is the runners avarge speed Ever since 1959 Cuba has had what type of government? Amber prefers to date men who are 20 years her senior. Although the women Jerome dates are approximately his age, they seem uncannily similar to his mother in some way. A psychoanalyst might view Amber's "Daddy" issues and Jerome's "Mommy" issues as reflecting fixation at the_____stage. find the values for a and b. Find the exact value of tan (-1740) What is the boiling point ofgold1064.43C2970C1313C3266C What are some informative speech ideas? Im supposed to present a speech and need a base topic :) BRAINLIEST ANSWER Twice the difference of a number and 4 is equal to three times the sum of the number and 6. Find the number. Whats the a chicken nugget Una pelota cae con una energa de 1 J y bota al llegar al suelo. Si tras el bote su energa es de 0,998 J, podemos afirmar que la energa se ha destruido? Janice already has $65 dollars in her savings account. If she puts $5 per week in her account, write an equation to find out how many weeks she must save to have at least $200 in her account.2 pointsYour answer Which of the following clauses states that full payment of damages to structures under the homeowners policy will be made only if the insurance equals 80 or more percent of the replacement cost of the structure and is carried on the property at the time of the loss?a. Coinsurance clause. b. Inflation rider clause. c. Reinsurance clause. d. Replacement clause What might happen to Salva without his uncle?A long walk to water chapter 9 plz don't guess HELPPPPPPPPPjonah earned $5 more than half of karens salary, k. which of the following expressions represents jonahs earnings.Jonah earned $5 more than half of Karen's salary, k. Which of the following expressions represents Jonah's earnings? one half(k + 5) one halfk + 5 one half + k + 5 one halfk > 5 During the industrial revolution many factory who adopted the assembly line Can someone answer this i need help Two Parallel lines are cut by a transversal and form a pair of alternate exterior angles. One angle measures (6x+5) and other measures (7x+4). Explain how to determine what those other angles actually measure. Please help its geometry f h(x) = 2x + 1, find h(2x-3).